Answer to Question #162526 in Optics for Carty

Question #162526

The 2 speakers of a boom box are 35.0 cm apart. A single oscillator makes the speaker vibrate in phase at a frequency of 2.00kHz. At what angles, measured from the perpendicular bisector of the line joining the speakers, would a distant observer hear (a) maximum sound intensity?

(b) minimum sound intensity?

(Speed of sound = 340m/s)


1
Expert's answer
2021-02-24T12:51:08-0500

Let's first find the wavelength of the wave from the wave speed equation:


"v=f\\lambda,""\\lambda=\\dfrac{v}{f}=\\dfrac{340\\ \\dfrac{m}{s}}{2\\cdot10^3\\ Hz}=0.17\\ m."

(a) Observer would hear the maximum sound intensity at the angles where there is a constructive interference of two sound waves:


"dsin\\theta=m\\lambda."

Then, from this equation we can find "\\theta":


"\\theta=sin^{-1}(\\dfrac{m\\lambda}{d})."

Finally, we get:


"\\theta_0=sin^{-1}(\\dfrac{0\\cdot0.17\\ m}{0.35\\ m})=0^{\\circ},""\\theta_1=sin^{-1}(\\dfrac{1\\cdot0.17\\ m}{0.35\\ m})=29^{\\circ},""\\theta_2=sin^{-1}(\\dfrac{2\\cdot0.17\\ m}{0.35\\ m})=76.3^{\\circ}."

For "m>2" there are no solutions, since "sin\\theta>1".

(b) Observer would hear the minimum sound intensity at the angles where there is a destructive interference of two sound waves:


"dsin\\theta=(m+\\dfrac{1}{2})\\lambda."

Then, from this equation we can find "\\theta":


"\\theta=sin^{-1}(\\dfrac{(m+\\dfrac{1}{2})\\lambda}{d})."

Finally, we get:


"\\theta_0=sin^{-1}(\\dfrac{(0+\\dfrac{1}{2})\\cdot0.17\\ m}{0.35\\ m})=14^{\\circ},""\\theta_1=sin^{-1}(\\dfrac{(1+\\dfrac{1}{2})\\cdot0.17\\ m}{0.35\\ m})=46.8^{\\circ},"

For "m>1" there are no solutions, since "sin\\theta>1".


Need a fast expert's response?

Submit order

and get a quick answer at the best price

for any assignment or question with DETAILED EXPLANATIONS!

Comments

No comments. Be the first!

Leave a comment

LATEST TUTORIALS
New on Blog
APPROVED BY CLIENTS